Einheitlicher Raumzeit-Übersetzungsoperator

Srednicki schreibt: Wir können dies ein wenig schicker machen, indem wir den einheitlichen Raumzeit-Übersetzungsoperator definieren

T ( A ) exp ( ich P μ A μ / )

Dann haben wir

T ( A ) 1 ϕ ( X ) T ( A ) = ϕ ( X A )

Wie erhalten wir die zweite Gleichung aus der ersten Gleichung?

Ich vermute, Herr Taylor (von Serienruhm) könnte dabei helfen ...?

Antworten (2)

Es ist unnötig, Hilbert-Raumzustände zu verwenden, um dieses Ergebnis formal abzuleiten; es folgt schnell aus einem nützlichen Ergebnis über das Matrix-Exponential (was sehr praktisch ist, wenn man Lie-Algebren studiert, was wir hier übrigens im Wesentlichen betrachten).

Lassen X beliebig sein N -von- N komplexe Matrix, dann definieren wir den linearen Operator A D X auf dem Vektorraum solcher Matrizen durch

A D X Y = [ X , Y ]
für alle N -von- N komplexe Matrizen Y . Hier [ , ] bezeichnet den Kommutator, der oft als adjungierter Operator bezeichnet wird. Dann haben wir folgendes Ergebnis :
e X Y e X = e A D X Y
Wenden wir dieses Ergebnis nun formal auf lineare Operatoren im Hilbert-Raum einer Quantenfeldtheorie an, dann erhalten wir
T ( A ) 1 ϕ ( X ) T ( A ) = e ich A μ P μ / ϕ ( X ) e ich A μ P μ / = e A D ich A μ P μ / ϕ ( X ) = k = 0 A D ich A μ P μ / k ϕ ( X ) k !
Jetzt verwenden wir die Tatsache, dass für jedes Feld Φ , wir haben
A D ich A μ P μ / ϕ ( X ) = ich A μ [ P μ , ϕ ( X ) ] = ich A μ ( ich μ ) ϕ ( X ) = A μ μ ϕ ( X )
Anwendung dieses Ergebnisses k mal und setzen es in die Reihenentwicklung für die oben geschriebene Exponentialfunktion ein, erhalten wir
T ( A ) 1 ϕ ( X ) T ( A ) = k = 0 ( 1 ) k k ! ( A μ μ ) k ϕ ( X )
Nun stellen wir einfach fest, dass die rechte Seite die Taylorentwicklung von ist ϕ ( X A ) . Explizit
ϕ ( X A ) = ϕ ( X ) A μ μ ϕ + 1 2 ( A μ μ ) 2 ϕ ( X ) + + ( 1 ) k k ! ( A μ μ ) k ϕ ( X ) +
und das ergibt das gewünschte Ergebnis.

Ich lese QFT aus Srednickis Buch. Im 2. Kapitel dieses Buches und im Spin-Halbteil dieses Buches wird die Gruppentheorie und Gruppenrepräsentationstheorie verwendet. Könnt ihr mir ein Buch empfehlen, aus dem ich das lernen kann?
Ich persönlich mag Brian Halls Lie Groups, Lie Algebras und Representations
Vielen Dank. Ich habe benn auf Ihre Antwort gewartet. Srednicki bringt mich um!
Dies könnte auch für Referenzen hilfreich sein . physical.stackexchange.com/questions/6108/…
Ich mag auch Group Theory in Physics von Wu-Ki Tung.

[EDIT] Angenommen, eine staatliche Basis | e ich , werden wir die folgende Notation für einen Zustand verwenden: | A ( X ) = ich A ich ( X ) | e ich

Ein Operateur Ö auftragen A gibt dann: Ö | A ( X ) = ich Ö A ich ( X ) | e ich

Zum Beispiel, μ | A ( X ) = μ A ich ( X ) | e ich

Anstatt mit Operatoren zu arbeiten, denke ich, dass es jetzt einfacher ist, mit Zuständen zu arbeiten | A ( X ) Und | B ( X ) wie zum Beispiel:

(1) | B ( X ) = Φ ( X ) | A ( X )

Dies gilt natürlich z X A , das ist:

(2) | B ( X A ) = Φ ( X A ) | A ( X A )

Wir wissen das:

P μ | A ( X ) = ich μ | A ( X ) .

Wir erhalten also:

T ( A ) 1 | A ( X ) = e ich P μ A μ / | A ( X ) = e A μ μ | A ( X ) = | A ( X A )

Die letzte Gleichheit ist einfach die Taylor-Reihe von | A ( X A ) bei X , das ist:

| A ( X A ) = | A ( X ) A μ μ | A ( X ) + 1 2 ! ( A μ μ ) 2 | A ( X ) + ( 1 ) N N ! ( A μ μ ) N | A ( X ) + .

Jetzt bewerben T ( A ) 1 zur Gleichung ( 1 ) , wir bekommen:

T ( A ) 1 | B ( X ) = T ( A ) 1 Φ ( X ) | A ( X ) .

Das ist:

T ( A ) 1 | B ( X ) = T ( A ) 1 Φ ( X ) T ( A ) T ( A ) 1 | A ( X ) .

Wir erhalten also:

| B ( X A ) = T ( A ) 1 Φ ( X ) T ( A ) | A ( X A ) .

Gleichung betrachten ( 2 ) , erhalten wir schließlich:

T ( A ) 1 Φ ( X ) T ( A ) = Φ ( X A )

neeeeeein! twistor59 hat versucht, OP zu dieser eigenen Lösung zu führen :(
@Trimok: Vielen Dank. Sie haben geschrieben: e a . | EIN ( x ) eein . | EIN ( x ) . Was bedeutet hier eine . ein . bedeuten?
@Ome: ein . ein . bedeutet ein μμAμμ . Ich habe eine Bearbeitung in der Antwort vorgenommen.
@Ome: Beachten Sie, dass mit Ihrer Konvention T ( a ) exp( - ich P μ ein μ /)T( ein ) exp( - d.hPμAμ/) , verwende ich dann die Konvention P μ | EIN ( x ) = ich μ | A ( x ) .Pμ| A ( x ) =ichμ| A ( x ) . . Aber andere Konventionen sind T ( a ) exp( ich P μ ein μ /)T( ein ) exp( ichPμAμ/) und P μ | EIN ( x ) = ich μ | A ( x ) .Pμ| EIN ( x ) =ichμ| A ( x ) . . Das Endergebnis ändert sich dadurch nicht.
@Trimok Was genau ist die Definition des Staates | EIN ( x ) | EIN ( x ) die du verwendest? Was ist insbesondere der mathematische Kontext Ihrer ersten Gleichung?
*mathematischer Inhalt, nicht Kontext; Verzeihung.
@Trimok: Vielen Dank. In der Antwort sehe ich: | EIN ( x ) | EIN ( x ) . Ich möchte hier wissen, wie es genau ist und möchte auch wissen, warum Sie ein x gesetzt habenX in Klammer nach AA ?
@Ome: Wichtig ist, den Unterschied zwischen den "Wellenfunktionen" A zu machenA und BB , und der Operator Φ ( x )Φ ( x ) . Φ ( x )Φ ( x ) ist kein Zustand, es ist keine Wellenfunktion, es ist ein Operator, der auf Zustände angewendet wird (wie P μPμ ist ein Operator, der auf Zustände angewendet wird). Die Transformationsgesetze von Zuständen und Operatoren sind also unterschiedlich. In Ihrem Beispiel handelt es sich um eine Übersetzungstransformation. Mit Ihren Konventionen ist die Transformation für einen Zustand SS ist T ( a ) 1 ST( ein)1S , während die Transformation für einen Operator OÖ ist T ( ein ) 1 O T ( ein )T( ein)1O T( ein )
@joshphysics: siehe meine vorherigen Kommentare zu Ome. AA ein beliebiger Zustand ist, und die Definition von BB ist einfach B = Φ AB = ΦA _ . Ich fand es einfacher, mit Zuständen zu arbeiten, weil die Operatoren P μPμ handeln Sie einfach auf sie.
@Trimok Mir ist die Definition von | immer noch unklar EIN ( x ) | EIN ( x ) . Wenn es sich einfach um einen Zustand im Hilbert-Raum der QFT handelt, warum fügen Sie ihm dann ein Raumzeit-Argument hinzu? Handelt es sich vielleicht um einen Zustand, der einer klassischen Feldkonfiguration A entsprichtA ? Außerdem verwendest du bei deiner Herleitung im Wesentlichen die folgende Tatsache: μ | EIN ( x ) = | μ EIN ( x ) μ| EIN(x)= |μEIN ( x ) . Können Sie dies bei einer beliebigen Definition des Staates begründen | EIN ( x ) | EIN ( x ) du benutzt?
@Ome: | EIN ( x ) | EIN ( x ) ist ein "KET"-Zustand in Dirac-Notation, also ein Vektor. Das ist dasselbe wie eine "Wellenfunktion" A ( x )Ein ( x ) . Sie können die Wellenfunktion A ( x ) zerlegenEin ( x ) auf irgendeiner Basis B iBich , mit EIN ( x ) = EIN ich ( x ) B ichEIN ( x ) = Aich( x )Bich . Dies ist dasselbe wie das Schreiben von | EIN ( x ) = EIN ich ( x ) | B ich| EIN(x)=Aich( x ) |Bich . Jetzt habe ich das x behaltenX Index, um zu bedenken, dass diese "KET" oder diese "Wellenfunktion" von Raum-Zeit-Koordinaten abhängt (hier ist "x" eine allgemeine Notation für Raum-Zeit-Koordinaten ( x 0 = t , x 1 = x , x 2 = y , x 3 = z ) )(X0= t ,X1= x ,X2= j,X3= z) ) . | EIN ( x ) | EIN ( x ) ist dasselbe wie eine "Wellenfunktion" A ( x , t )Ein (X⃗ , t ) .
@joshphysics: Ich habe in einem früheren Kommentar einen Fehler gemacht, den ich lösche und durch einen anderen Kommentar zu Ome ersetze (siehe oben). Der richtige Weg ist, eine Basis | zu betrachten B ich >|Bich> mit | EIN ( x ) >= EIN ich ( x ) | B ich >| EIN(x)> =Aich( x ) |Bich> , also μ | EIN ( x ) >= μ EIN ich ( x ) | B ich >μ| EIN(x)> =μAich( x ) |Bich>
@Trimok Versuchen Sie zu sagen, dass bei einer klassischen Feldkonfiguration AA , gibt es einen Zustand im Hilbert-Raum, der dieser Konfiguration entspricht? Wenn ja, können Sie das begründen? Ich bin ehrlich, ich verstehe nicht, wie Ihre Ableitung in der Antwort mathematisch korrekt ist.
@Ome: Ich habe am Anfang der Antwort eine Bearbeitung vorgenommen.
@joshphysics: Ich verstehe deinen Punkt nicht. | A ( x ) >| A(x)> ist ein Zustand, und die Auswahl einer Basis | B ich >|Bich> , dieser Zustand hat die Koordinaten A i ( x )Aich( x ) auf dieser Grundlage.
@Trimok Warum haben die Koeffizienten in der Expansion eines bestimmten Zustands Raumzeitargumente? Gegeben einige Grundlage | e ich|eich für einen Hilbertraum kann man einfach einen beliebigen Zustand | schreiben ψ | ψ als ich c ich | e ichichCich|eich ; keine Raumzeit-Argumente notwendig. Ich versuche, die Raumzeitargumente zu verstehen, die Sie im Zusammenhang mit QFT schreiben.
@joshphysics: Wenn ich einen Zustand nehme, der nicht von x abhängtX , und wenn ich einen Operator anwende, der von x abhängtX Auf diesem Zustand erhalte ich einen Endzustand, der von x abhängtX